Warum werden Antikommutatoren bei der Quantisierung von Dirac-Feldern benötigt?

Warum wird der Antikommutator eigentlich bei der kanonischen Quantisierung des freien Dirac-Feldes benötigt?

Eine Diskussion dieser Frage findet sich in Peskin und Schroeder, An Introduction to QFT, Abschnitt 3.5.
Kurze Antwort: Auf diese Weise folgen Dirac-Teilchen der Pauli-Fermi-Ausschlussregel

Antworten (2)

Der elementarste Grund ist, dass der Dirac-Feld-Hamilton-Operator nur dann nach unten begrenzt ist, wenn Sie Antikommutierungsbeziehungen auf die Erzeugungs-/Vernichtungsoperatoren anstelle von Kommutatoren verwenden. Eine freie Quantenfeldtheorie mit nach unten unbegrenzter Energie hat kein stabiles Vakuum.

Es ist am einfachsten, dies in zwei Dimensionen zu demonstrieren, wo es keine Polarisierungsprobleme gibt.

Lehrreiches 2D-Beispiel

In zwei Dimensionen (ein Raum einmal) gibt es ein schönes dimensionsreduziertes Analogon, nämlich das sich nach rechts bewegende (notwendigerweise masselose) Majorana-Weyl-Fermion (das Argument funktioniert auch mit 2d-Dirac-Fermionen mit zwei Komponenten, aber dies ist das einfachste Fall). Dies ist ein Einzelkomponentenfeld ψ die der Bewegungsgleichung gehorcht

( t x ) ψ = 0

Diese einfache Gleichung wird aus der 2d-Dirac-Gleichung unter Verwendung der (reellen Konvention, explizit reellen) 2d-Dirac-Matrizen (0,1;-1,0) und (0,1;1,0) abgeleitet, die sind γ 0 = σ x und γ 1 = ich σ j . Sie quadrieren zu 1 bzw. -1 und sind antikommutierend, sodass sie den 1 + 1-dimensionalen metrischen Tensor reproduzieren. Das γ 5 analog, die ich anrufen werde Γ um unterschiedliche Dimensionen aufzunehmen, ist in dieser expliziten Darstellung diagonal, und Γ = σ z .

Die beiden Eigenvektoren von Γ sich unabhängig durch die 2d masselose Bewegungsgleichung ausbreiten

γ ich ich ψ = 0

Und weiter, weil die γ Matrizen reell sind, ist dies eine Majorana-Darstellung (die meisten Physiker schreiben die Dirac-Gleichung mit einem i-Faktor vor die Ableitung, so dass die Dirac-Matrizen für eine Majorana-Darstellung rein imaginär sind. Ich verwende dafür eine Mathematiker-Konvention, weil Ich mag es, wenn die Bewegungsgleichungen real sind. Andere mögen es, wenn der k-Raum-Propagator keine Faktoren von i im k-Teil hat. Leider haben sich die Physiker nie auf eine eindeutige vernünftige Konvention festgelegt – jeder hat seine eigene bevorzugte Art, Dirac zu schreiben Matrizen). Es ist also sinnvoll in der Bewegungsgleichung einzuschränken ψ hermitesch zu sein, da ihr hermitesches Konjugat genau derselben Gleichung gehorcht.

Das Feld hat also eine k-Zerlegung

ψ ( x ) = a k e ich k x ich k t d k

Und die Realitätsbedingung (Hermitizität) sagt dir das a ( k ) = a ( k ) (Man sollte sagen, dass die Normalisierung der a Operatoren Erweiterung ist konzeptionell nicht ganz trivial--- die a 's sind sowohl relativistisch als auch nichtrelativistisch normiert, da die Spinorpolarisation w Faktor hebt den Masse-Schalen-Hyperbelfaktor auf, so dass die dk-Integration durch nichts gewichtet wird, es ist nur das normale Kalkül-Integral mit einheitlichem Maß)

Ein Operator mit bestimmter Frequenz, der sich zeitlich entsprechend (Heisenberg-Bild) entwickelt

t Ö = ich ω Ö

Hat die Eigenschaft, dass es sich um einen Erhöhungsoperator handelt --- mit diesem Operator addiert ω zur Energie. Wenn ω ist negativ, a ist ein Vernichtungsoperator. Die Bedingung, dass das Vakuum stabil ist, besagt, dass alle Vernichtungsoperatoren 0 ergeben, wenn sie auf den Vakuumzustand einwirken.

Aber beachten Sie, dass die Frequenz in der Erweiterung von ψ wechselt das Vorzeichen bei k = 0 . Dies kam von der Linearität des Dirac-Hamilton-Operators in den Impulsen. Es bedeutet, dass der Betreiber a k wirkt, um die Energie für k > 0 zu erhöhen, aber wirkt, um die Energie für zu senken k < 0 . Dies bedeutet, dass die k > 0 Betreiber erstellen, und die k < 0 Betreiber vernichten, damit der richtige Weg zu a ( k ) sind Erstellungsoperatoren, während die k < 0 Operatoren sind Vernichtungsoperatoren.

Der Energieoperator zählt die Anzahl der Impulsteilchen k und multipliziert mit ihrer Energie:

H = k > 0 k a ( k ) a ( k ) d k

Und dies ist offensichtlich kein lokaler Operator, sondern nur integriert über k>0 definiert. Um es zu einem lokalen Operator zu machen, müssen Sie die Integration auf alle k erweitern, aber dann haben die negativen k- und positiven k-Beiträge entgegengesetzte Vorzeichen und sie müssen gleich sein. Um dies zu arrangieren, müssen Sie Antikommutierungsbeziehungen nehmen

{ a ( k ) , a ( k ) } = ich δ ( k k ' )

Und dann

H = 1 2 k a ( k ) a ( k ) = ψ ( x ) ich x ψ ( x ) d x

Beachten Sie, dass dies so aussieht, als wäre es eine perfekte Ableitung, und es wäre, wenn ψ waren keine Anti-Pendel-Menge. Für Anti-Pendel-Mengen gilt:

x ψ 2 = ψ x ψ + x ψ ψ

Was wegen der Antikommutierung Null ist.

Tiefere Gründe

Obwohl dies wie eine zufällige Eigenschaft aussieht, dass die Energie ohne Antikommutatoren negativ war, ist sie es nicht. Der tiefere Grund wird mit der euklidischen Feldtheorie unter Verwendung eines Feynman-Schwinger-Formalismus erklärt, aber dies erfordert ein Verständnis der euklidischen und pfadintegralen Versionen von Antipendelfeldern, was erfordert, dass man sich mit Antipendelgrößen wohlfühlt, was eine Motivation erfordert. Also ist es am besten, zuerst den seichten Grund zu lernen.

Warum brauche ich alle Beziehungen, um gegen das Pendeln zu sein? Ich kann davon ausgehen, dass 2 von ihnen gegen das Pendeln sind, aber der dritte, dh die Beziehung zwischen dem Schöpfungs- und dem Vernichtungsoperator, ist pendelnd und behält dennoch den Ausschluss von Pauli bei. physical.stackexchange.com/q77384

Angenommen, die Frage lautet, warum Antikommutatoren und nicht Kommutatoren, gemäß Davids Kommentar:

Wenn ich die Eigenschaften der Dirac-Teilchen mit einem allgemeinen Index bezeichne a - diese können Spins, Impulse beinhalten - dann, wenn ich einen Zwei-Teilchen-Zustand erzeuge, wobei Teilchen 1 hat a 1 und Teilchen 2 a 2 dann wird der Zustand durch Anwenden von Erstellungsoperatoren in der Reihenfolge angegeben

| a 1 a 2 = b a 1 b a 2 | 0 >
Wenn wir sie umgekehrt erstellen:
| a 2 a 1 = b a 2 b a 1 | 0 >
dann ist es leicht zu sehen, wenn die b Antikommutierungsbeziehungen befolgen
| a 2 a 1 = | a 1 a 2
dh die Theorie reproduziert natürlich die Fermi-Statistik, wie Sie es für Dirac-Teilchen mit Spin 1/2 wünschen würden.

Ja, das stimmt, aber ich denke, das OP wollte einen Beweis für die Spin-Statistik. Den Beweis habe ich im einfachsten Fall geführt, 2d Majorana Weyl Spinors.
Ja, ich war mir nicht sicher, auf welche Ebene die Frage abzielte - ich entschied mich für "warum Antikommutatoren statt Kommutatoren". Sie haben die schwierigere Frage beantwortet (+1 für einen einfachen Beweis)
Ok, +1 auch für dich.